Marco de referencia giratorio

¿Puedes ayudarme a hacer esto?

Dos marcos de referencias S y S tener un origen común O y S gira con velocidad angular constante ω con respecto a S .
un aro cuadrado A B C D está hecho de alambre fino y liso y tiene longitud lateral 2 a . El aro es horizontal y gira con velocidad angular constante. ω sobre un eje vertical que pasa por A . Una pequeña cuenta que puede deslizarse sobre el alambre está inicialmente en reposo en el punto medio del lado B C . Elija ejes relativos al aro y deje y sea ​​la distancia de la cuenta al vértice B en el lado B C . Escriba el vector de posición de la cuenta en su marco giratorio. Muestra esa
y ¨ ω 2 y = 0 utilizando la expresión de la aceleración. Por lo tanto, encuentre el tiempo que tarda la cuenta en llegar a un vértice. C .

mostré eso d 2 r d t 2 = ( d 2 r d t 2 ) + 2 ω × ( d r d t ) + ω × ( ω × r ) dónde indica que está hecho en marco giratorio. r es el vector de posición de un punto PAG medida desde el origen.

lo tengo
r = r porque θ i + y j
r = ( r ˙ C o s θ r θ ˙ pecado θ ) i + y ˙ j
r = ( r ¨ C o s θ r ˙ θ ˙ s i norte θ r ˙ θ ˙ pecado θ r θ ¨ pecado θ r θ ˙ 2 C o s θ ) i + y ¨ j
ω × r = ω y ˙ i + ( ω r ˙ porque θ ω r θ ˙ pecado θ ) j
ω × ( ω × r ) = ω 2 r porque θ i ω 2 y j

Supongo que ahora tengo que escribir la segunda ley de Newton, pero no sé qué fuerzas tengo en este movimiento.

Tus ecuaciones son bastante difíciles de seguir. Qué es θ en la ecuacion uno? y como escribiste r = r porque θ i + y j ? Tenga en cuenta que y es la distancia de la cuenta desde B y no del eje x, por lo que creo que esta ecuación es incorrecta. Tal vez su enfoque del problema no es correcto. Conozco otra forma de solucionar esto, si quieres te la publico.
θ es un ángulo entre el vector de posición y el eje x en el marco giratorio. debería elegir hachas X y y de manera que queden a lo largo de los lados del cuadrado. elegí eso X se acuesta A B y y se acuesta A C . Sí, por favor, escribe tu solución.

Respuestas (1)

Ahora veo el problema con tu ecuación.

al diferenciar r = r porque θ i + y j , has considerado r ser constante, lo cual es incorrecto.
r es dado por

r = yo 2 + y 2
dónde yo es la longitud del lado del cuadrado. Entonces r cambiará con y , y tendrás que diferenciar r también.
¡Aquí es donde las matemáticas se ponen feas y disuasorias!

Para evitar eso, lo que podemos hacer es observar que en el marco de rotación, la perla experimentará una fuerza centrífuga hacia afuera. Esta fuerza tendrá una componente a lo largo B C . Ese componente se puede escribir como (tomaré prestadas sus variables)

F B C = metro ω 2 r pecado θ
F B C = metro ω 2 yo 2 + y 2 y yo 2 + y 2

Así, dividiendo por metro en ambos lados se obtiene

y ¨ = ω 2 y

Tenga en cuenta que esto es lo mismo que aplicar d 2 r d t 2 = ( d 2 r d t 2 ) + 2 ω × ( d r d t ) + ω × ( ω × r ) . Es solo que este enfoque es más específico para el problema (¡y mucho más fácil también!).

El problema es que TENGO que usar esta fórmula para la aceleración. Ok, lo diferenciaré nuevamente teniendo en cuenta quer = r ( t ) y voy a ver que me sale. Solo dime esto: ¿Qué fuerzas actúan sobre una cuenta? Gravitacional y este que escribiste? ¿Son ambos perpendiculares al eje vertical? Si lo son, puedo multiplicar la expresión de la segunda ley de Newton con Isdr⃗ dθ Llegar0 en el lado derecho.
La ley de Newton no sería de mucha ayuda aquí. Esto se debe a que el bucle está restringido por algún agente externo para mantener una velocidad angular constante. Ahora bien, esta fuerza seguirá variando de acuerdo con el momento cambiante de inercia del sistema, y ​​variará de tal manera que el bucle mantenga una velocidad angular constante. Las ecuaciones de movimiento que tienes serán suficientes para resolver este problema.
Si aún no está satisfecho, puede pensarlo de esta manera. Todo lo que le dará la ecuación de la segunda ley de Newton es una descripción del movimiento de las cuentas, que ya tiene con usted. Solo diferencia la ecuación tomandor como variable y debería obtener su respuesta.
Corregí expresiones en cuestión. ¿Por qué no hay fuerza gravitatoria en tu respuesta? ¿Por qué observas solo la fuerza centrífuga?
Lo tengo. La fuerza gravitatoria no tiene componente a lo largoy -eje.
¿Estás de acuerdo en que si tomo
r⃗ = r porqueθi⃗ + yj⃗ 
como correcto, significaría quer porqueθ sería igual a2 un desdey es la distancia de la cuenta al vérticeB ? Si la cuenta no está al ladoB C esta ecuación no será aceptable. En ese caso la ecuación correcta sería
r⃗ = r porqueθi⃗ + r pecadoθj⃗ 
.
Entonces, si observo solo el caso cuando la cuenta está en el costadoB C , Puedo escribir
r⃗ = 2 uni⃗ + yj⃗ 
r⃗ ˙=y˙j⃗ 
r⃗ ¨=y¨j⃗ 
ω⃗ ×r⃗ ˙= ωy˙i⃗ 
ω⃗ × (ω⃗ ×r⃗ ) = ω yi⃗ + 2 un ωj⃗ 
d2r⃗ dt2= ( 2 wy˙+ 2 unω2)i⃗ + (y¨ω2y)j⃗ 
solo puedes escribirr⃗  en el marco giratorio comor⃗ = 2 uni⃗ + yj⃗  , En el marco inercial tendrías que usarr⃗ = r porqueθi^+ r pecadoθj^
Pero si observo sólo el caso cuando la cuenta está en elB C lado y si usod2r⃗ dt2= (d2r⃗ dt2)+ 2ω⃗ × (dr⃗ dt)+ω⃗ × (ω⃗ ×r⃗ ) , puedo escribirr⃗ = 2 uni⃗ + yj⃗  ?r⃗ = 2 uni⃗ + yj⃗  es el vector de posición en un marco giratorio.
si puedes escribirr⃗  de esa manera
En ese caso tengo
d2r⃗ dt2= ( 2 ωy˙2 unω2)i⃗ + (y¨ω2y)
¿Qué hacemos ahora con esto? ¿Cómo puedo demostrar que no hay fuerza activa en la dirección dej⃗  si hay una componente de fuerza centrifuga?
¿Es posible que la fuerza centrífuga ya esté contenida en la expresión parametrod2r⃗ dt2 comometroω⃗ × (ω⃗ ×r⃗ ) ?